RHIA MOCK EXAM

Pataasin ang iyong marka sa homework at exams ngayon gamit ang Quizwiz!

a 335-bed hospital opened a new wing on June 1 of a nonleap year, increasing its bed count to 350 beds. the total bed count days for the year at the hospital was a. 122, 275 b. 125, 485 c. 127, 750 d. the answer cannot be calculated with the information provided

a. 125, 485

use the information on errors in indexing of scanned material that you have collected and presented in the table below to answer the question referring again to the data collected on scanning errors, if you want to work on the type of material with the highest volume, you will work on problems with a. consultation reports b. lab slips c. correspondence d. other

b. lab slips

your HMO manager has requested a report on the number of patient visits per year for preschool children. which of the age groupings below will you use for your report. a. 0-1 year, 1-2 years, 2-3 years, 3-4 years, 4-5 years b. less than 12 months, 12-24 months, 25-37 months, 38-50 months, 51-63 months. c. greater than 12 months, 12-24 months, 25-37 months, 38-50 months, less than 51 months d. 0-2 years, 3-4 years, 5 years

b. less than 12 months, 12-24 months, 25-37 months, 38-50 months, 51-63 months.

a patient initially consulted with dr. vasseur at the request of dr. meche, the patient's primary care physician. dr. vasseur examined the patient, prescribed medication, and ordered tests. additional visits to dr. vasseur's office for continuing care would be assigned from which e/m section? a. office and other outpatient services, new patient b. office and other outpatient services, established patient c. office or other outpatient consultations, new or established patient d. confirmatory consultations, new or established patients

b. office and other outpatient services, established patient

in a research study that includes a patient questionnaire, five of the questions will be answering using the following scale: 1-strongly agree 2-disagree 3-no opinion 4-agree 5-strongly agree the data collected using this scale are called a. cardinal data b. ordinal data c. nominal data d. continuous data

b. ordinal data

the transiptionists have collected data on the number and types of problems with the dictation equipment. the best tool to display the data they collected is a a. flowchart b. pareto chart c. gantt chart d. pert chart

b. pareto chart

the credentialing committee discovered the problems with dr. hernandez's hip replacements by a. sampling and reviewing dr. henandez's patient records b. reviewing dr. hernandez's physician profile c. querying the national physician data bank d. interviewing the chief of surgery

b. reviewing dr. hernandez's physician profile

community hospital reported an average LOS in December of 3.7 days with a standard deviation of 23. this information indicates that a. there was a small variation in the LOS at the community hospital b. there was a large variation in the LOS at community hospital c. most of the patients at community hospital stay 3 to 4 days d. patients stay longer at community than at most hospitals

b. there was a large variation in the LOS at community hospital

you are providing an educational session to new hires at your hospital. you tell the new employees that hospital records may be used as evidence in court even though hearsay laws bar the use of most evidence that does not represent personal knowledge of the witness. that is because the hospital record a. is written rather than spoken b. was kept in the regular course of business c. has not been tampered with in any way d. is accurate and complete

b. was kept in the regular course of business

you are heading a research study that includes a patient questionnaire. five of the questions will be answered using the following scale: a. stacked bar graph b. pie chart c. frequency table d. frequency polygon

c. frequency table

a patient's husband slipped and fell in your HIM reception are and now he is suing the facility. you have to prepare detailed written answers to a long list of questions and send them to your hospital attorney. you will spend the afternoon working on a. affidavits b. allocutions c. interrogatories d. depostions

c. interrogatories

staging a. refers to the monitoring of incidence and trends associated with a disease b. is continued medical surveillance of a case c. is a system for documenting the extent or spread of cancer d. designated the degree of differentiation of cells

c. is a system for documenting the extent or spread of cancer

during the work sampling of a file clerk's activity, it is noted that the employee is speaking on the telephone during 76 of 300 observations. how much of the employee's time is spent on the phone if the employee works 7 hours a day? a. 1.77 hours b. 3.28% c. 3.94 hours d. 9.2%

1.77 hours

Mary is 6 weeks postmastectomy for carcinoma of the breast. she is admitted for chemotherapy. What is the correct sequencing of the codes C50.911 Malignant Neoplasm of the Breast Z85.3 Personal History of malignant neople of breast Z51.11 Encounter for antineoplastic chemotherapy Z08 Follow-up exam after surgery a. Z51.11, C50.911 b. Z51.11, Z85.3 c. Z08, Z51.11 d. Z85.3

A. Z51.11, C50.911

AMERICAN ASSOCIATION FOR MEDICAL TRANSCRIPTION, COMMITTED TO THE CREATION OF QUALITY HEALTH RECORD REPORTS FROM DICTATION BY HEALTHCARE PROVIDERS.

AAMT

A technical safeguard that involves unique user ID, emergency access procedures, automatic log-off, and encryption and decryption of data

Access controls

Keeping track of disclosures of patient information

Accounting of disclosures

Covers policies, procedures, contracts, plans for security, and contingency plans

Administrative safeguards

Used to develop information about groups of patients

Aggregate data

A patient's name identifies the patient record

Alphabetic Health Record Identification

Health care provided by methods other than the traditional inpatient care, including home health, ambulatory, hospice, and other types of health care.

Alternate Delivery Systems

Largest association for medical trasncription

American Association for Medical Transcription (AAMT)

Supports information technology professionals to improve health care

American Medical Informatics Association (AMIA)

Physician harm to a person

Assault

SOAP what does A stand for?

Assessment (combine subjective and objective to make a conclusion)

A report that shows the total dollar amounts in accounts at a specific point in time

Balance sheet

When an organization has acquired the "best" products from various vendors

Best-of-breed

Virtually all applications are provided by a single vendor

Best-of-fit

Provide accessible information about births and deaths

Births and Deaths Register

CENTERS FOR DISEASE CONTROL, CONCERNED WITH COMMUNICABLE DISEASES AND ENVIRONMENTAL HEALTH.

CDC

System that monitors all types of cancer diagnosed or treated in an institution

Cancer or Tumor Registry

The management of, responsibility for, or attention to the safety and well-being of another person or other persons

Care

Individual who is receiving professional services

Client

Structured plans of care

Clinical Pathways

Systematically developed statements used to assist provider and patient decisions about appropriate health care for specific clinical circumstances

Clinical Practice Guidelines

Communication is possible only on one vendor's system

Closed System Architecture

Individuals divided by street

Cluster random sample

The most complex type of access that is based on who the person is, what role they are playing, and what they can access

Context-based access

Legally enforceable agreements between two or more individuals

Contract

Prohibits conduct considered injurious to society as a whole and provides for punishment of those found to have engaged in such conduct

Criminal Law

Display goals for patients and provide the corresponding ideal sequence and timing of staff actions to achieve those goals with optimal efficiency

Critical Paths

DATA ELEMENTS FOR EMERGENCY DEPARTMENT SYSTEMS. UNIFORM SPECIFICATIONS FOR DATA ENTERED IN EMERGENCY DEPARTMENTS PATIENT RECORDS.

DEEDS

Associations between entities

Database Relationship

Records filed in multiple locations

Decentralized Filing

American College of Surgeons (ACS) was established to...?

Develop a system of hospital standardization to improve patient care and recognize hospitals that had the highest ideals

List of diseases and conditions according to the classification system used in the facility

Disease Index

Heath Care Quality Improvement Act (1986)

Established the NPDB

Third party payers and/ or patients issue payments to health care providers based on charges assigned to each service performed for each patient

Fee for Service

What did the Patient Self-Determination Act of 1990 establish?

Gave patients the right to set advance directives

(Number of deaths of inpatients/Number of discharges + deaths) x 100

How do you calculate gross death rate?

(Number of infections for a given period/Number of discharges + deaths) x 100

How do you calculate hospital-acquired infection rate?

What did the Flexner Report of 1910 do?

Identify serious problems and inconsistencies in medical education

When should Discharge Summary be documented?

Immediately after discharge of patient

When should Operative Report be documented in record?

Immediately following surgery

A number of new cases occurring in a given time period

Incidence rate

A statement that summarizes an organization's revenue and expense accounts

Income statement

Factual data that have been collected, combined, analyzed, interpreted, and converted into a form that can be used for a specific purpose

Information

Documentation from various sources organized in strict chronological or reverse chronological order

Integrated Health Records

Supports national associations and health record professionals to improve health records

International Federation of Health Record Organizations (IFHRO)

Promotes informatics in healthcare and bio-medical research

International Medical Informatics Association (IMIA)

Multiple devices connected via communications media and located in a small geographical area

Local Area Network (LAN)

MEDICARE RESTRUCTURING OF REIMBURSEMENT OF INPATIENT HOSPITAL ADMISSIONS.

MS-DRG

Identifies all patients admitted to a health care facility for treatment, along with their identifying information

Master Patient Index (MPI)

NATIONAL COMMITTEE FOR QUALITY ASSURANCE, ACCREDITING AGENCY FOR MANAGED CARE ORGANIZATIONS.

NCQA

NATIONAL COMMITTEE ON VITAL AND HEALTH STATISTICS, ADVISORY BODY ON HEALTH DATA, STATISTICS, AND NATIONAL HEALTH INFORMATION POLICY.

NCVHS

Chronological list of patient's identification numbers issued to patients

Number Index

A number identifies the patient record

Numeric Identification

Admission and Discharge Register -- How long retained and in what order?

Permanent in chronological order

All cases, old and new, occurring in a given time period

Prevalence rate

The right of an individual to be left alone

Privacy

American Hospital Association (AHA) was established to...?

Promote public welfare by providing better health care in hospitals

Deals with relationships between private parties and the government (criminal law and government regulations)

Public Law

Review of record to ensure that standards are met and determine the adequacy of entries documenting the quality of care

Qualitative Analysis

A review of health record to determine its completeness and accuracy

Quantitative Analysis

Created to monitor various diseases and health problems with different goals and objectives

Registry

Database management system in which data are organized and managed as a collection of tables

Relational Model Database

A part of the System Development Life Cycle (SDLC) design phase

Request for proposal (RFP)

for reimbursement of physician services of beneficiaries covered under Medicare Part B

Resource-Based Relative Value Scale (RBRVS)

Based on what role a person plays (ex: nurse, physician, etc.)

Role-based access

What are records in a database table?

Rows

A new number is assigned to the patient for each new encounter at the facility, but the former records are brought forward and filed under the new number

Serial-Unit Numbering

Documents organized into sections according to the provider's and departments that provide treatment (lab together, rad. together, clinical notes together)

Source-Oriented Health Record

Individuals sorted by subgroups like male and female

Stratified random sample

What is another word for file or entity

Table

Numeric filing is divided into three parts and is read from right to left instead of left to right

Terminal Digit Filing

Who owns the information within the health record?

The patient

Third party payers and/ or patients pay health care providers after services have been rendered

Traditional Fee-For-Service

Uniform collection of data on inpatients

Uniform Hospital Discharge Data Set (UHDDS)

Mainframes, minicomputers, microcomputers

What are the 3 types of computers?

Minimum of 10 years

What does AHA recommend for record retention?

A computer network that connects separate institutions across a large geographical area

Wide Area Network (WAN)

When should Verbal Orders be cosigned?

Within 24 hours

use the information on errors in indexing of scanned material that you have collected and presented in the table below to answer the question if you want to begin with the type of material that has the highest error rate, you will start by working on problems with a. consultation reports b. lab slips c. correspondence d. other

a. consultation reports

the special form or view that plays the central role in planning and providing care at skilled nursing, psychiatric, and rehabilitation facilities is the a. interdisciplinary patient care plan b. medical history and review of systems c. interval summary d. problem list

a. interdisciplinary patient care plan

as your meeting with the clerical staff on the stat report continues, one clerk suggests a possible reason for the delays i a lack of training concerning the nature of stat reports. on the cause and effect diagram, this would most appropriately be listed under a. personnel b. equipment c. materials d. methods

a. personnel

access to radiologic images has been imporved through the use of which of the following? a. LOINC b. PACS c. EDMS d. CPOE

b. PACS

the pharmacy and therapeutic committee has asked you to find out more about a computerized order entry system that calculates drug dosages based on patient parameters (weight, age, etc.) and even suggests the best drug given the patient's diagnosis and current treatment. the committee is asking for information on a. application system b. clinical decision support system c. ordering system d. practice parameters system

b. clinical decision support system

according to CPT, a biopsy of the breast that involves removal of only a portion of the lesion for pathologic examination is a. percuatneous b. incisional c. excisional d. punch

b. incisional

which of the following are considered a (late effect) sequela regardless of time? a. congenital defect b. nonunion c. nonhealing fracture d. poisoning

b. nonunion

the state is considering the closure of the Arcadia Hospital. in reviewing the hospital statistics, which indicator will best help state officials determine whether closure is warranted? a. daily census b. percentage of occupancy c. inpatient service days d. average length of stay

b. percentage of occupancy

patients with diabetes participated in a study to determine the effectiveness of a new drug. the drug was taken at bedtime. the drug company expected patients taking glucodown to have a normal early morning fasting blood sugar level. the null hypothesis for the study follows there will be no difference in fasting early morning blood sugar levels between patients taking glucodown and patients take a placebo. half the patients were given glucodown. half were given a placebo. early morning fasting blood sugar levels are reported below. Based on the information, you would expect the researchers to a. accept the null hypothesis b. reject the null hypothesis c. restate the null hypothesis d. draw no conclusions

b. reject the null hypothesis

a number of key elements for your facility's computerized patient record are still input by clerical staff from handwritten data entry sheets. you are concerned about the transfer of data. if the vital signs stored in the database are not what were originally recorded, the impact on patient care could be severe. you are concerned about the a. stability of the data b. validity of the data c. legitimacy of the data d. reliability of the data

b. validity of the data

provide the CPT code for anesthesia servcies for the transvenous insertion of a pacemaker 00530- anesthesia for permanent transvenous pacemaker insertion 00560- anesthesia for procedures on heart, pericardial sac, and great vessels of chest; without pump oxygenator 33202- insertion of epicardial electrodes; open incision 33206- insertion or replacement of permanent pacemaker with transvenous electrodes; atrial a. 00560 b. 33202, 00530 c. 00530 d. 33206, 00560

c. 00530

authentication is one of the components necessary to produce a legal document in an EHR. this means a. tracking changes in the EHR system. b. establishing access controls for individual employees c. creating audit trails d. identifying who created a document and when

d. identifying who created a document and when

Serves needs of health care chief information officers and advocates for more effective use of information management in health care

College of Healthcare Information Management Executives (CHIME)

What are fields in a database table?

Columns

Consists of principles that have evolved over time from court decisions resolving controversies

Common Law

the coding supervisor tends to deal with issues as they come up, prioritizing only when problems are pressing or appear to be important to upper management. this crisis manager is particularly weak in which management function? a. planning b. organizing c. controlling d. budgeting

a. planning

Your facility is engaged in a research project concerning patients newly diagnosed with type 2 diabetes. the researchers notice older patients have a longer length of stay than younger patients. they have seen a a. positive correlation between age and length of stay b. negative correlation between age and length of stay c. causal relationship between age and length of stay d. homologous relationship between age and length of stay

a. positive correlation between age and length of stay

a run or line chart would be most useful for collecting data on a. waiting time in the pediatrics clinic b. patient satisfaction with the food c. delays in scheduling elective surgical procedures d. medication errors and their causes

a. waiting time in the pediatrics clinic

Individuals selected from a list by drawing every nth unit

Systematic random sample

An injury or wrong committed against an individual or his property. One party asserts wrongful conduct on part of the other and seeks compensation for harm suffered

Tort

What did the Tax Equity and Fiscal Responsibilities Act (TEFRA) do?

Established a mechanism for controlling the cost of the Medicare program and set limits on reimbursement and required the development of the prospective payment system

Disadvantage of Integrated Health Record?

Difficult to compare similar information (ex. lab results or oncology information)

A system that combines the financial and clinical aspects of healthcare and uses a group of providers to furnish comprehensive healthcare services across the continuum of care

Integrated delivery system (IDS)

Divided into four parts: database, problem list, initial plan, progress notes (SOAP)

Problem-Oriented Health Record

List of surgical and procedural codes

Procedure or Operation Index

Hospitals organized around product line categories (obstetrics, rehabilitation, cardiology)

Product Line Management

Clear definitions provided so users know what data means, each data element should have clear meaning and accepted values

Data Definition

Physical equipment that makes up computers and computer systems

Hardware

The graph has the tail off to the left

Negatively skewed

Data is organized in tree's according to relationships (many to many)

Network Database Model

a 19-year-old former patient faxes a request to your facility requesting the release of his medical records of all episodes of care to the army. the release of information clerk should a. send the records as requested b. inform the young man that specific reports must be identified in his request c. send a letter informing him that faxed requests are not accepted d. deny the request

a. send the records as requested

Touching a person's body without permission

Battery

in preparation for conversion to a computerized patient record, a committee at your facility is defining each of the data elements in a patient record to determine which elements should be required and to set parameters for each element. the committee is working on the data a. edits b. reasonableness c. dictionary d. feasibiltiy

c. dictionary

your HIS department receives and authorization for sara may's medical history to be sent to her attorney, but the expiration date noted on the authorization has passed. what action is appropriate according to hipaa privacy rules? a. do not honor because the authorization is invalid. b. contact the patient to get permission to respond c. contact the attending physician for permission to respond d. honor the authorization since the patient obviously approves of the release

a. do not honor because the authorization is invalid

The attributes and values of data should be defined at the correct level of detail

Data Granularity

Plan or pattern for an information system, including the database structure, known as a conceptual model, and the translation of the concept to the computer, known as the physical model

Data Model

The data are meaningful to the performance of the process or application for which they are collected

Data Relevance

Determined by how the data are being used and their context

Data Timeliness

Collection of bits make up a byte

Database Character

Someone who works 32 to 40 hours each week

Full time employee

HEALTH PLAN EMPLOYER DATA AND INFORMATION SET, STANDARDIZED SET OF PERFORMANCE MEASURES DESIGNED TO ALLOW PURCHASE/CONSUMERS OF MANAGED CARE PLANS TO COMPARE PERFORMANCE.

HEDIS

HEALTH INFORMATION MANAGEMENT AND SYSTEMS SOCIETY, FOCUSES ON OPTIMAL USE OF HEALTH CARE INFORMATION TECHNOLOGY AND MANAGEMENT SYSTEMS FOR THE BETTERMENT OF HUMAN HEALTH.

HIMSS

HEALTH INSURANCE PORTABILITY AND ACCOUNTABILITY ACT OF 1996. LEGISLATION IN 1996 WHOSE FOCUS IS HEALTH INSURANCE AND HEALTH INFORMATION.

HIPAA

Provides leadership in health care for the management of technology management systems

Health Care Information Management Systems Society (HIMSS)

Managed health care that integrates health care delivery with insurance for healthcare

Health Maintenance Organizations (HMO's)

NATIONAL ELECTRONIC DISEASE SURVEILLANCE SYSTEM, WEB-BASED STRATEGY INTENDED TO MONITOR/ ASSESS DISEASE TRENDS, GUIDE PREVENTION/INTERVENTION PROGRAMS, INFORM PUBLIC HEALTH MAKERS, IDENTIFY ISSUES OF CONCERN.

NEDSS

One physical topology of a network and uses a central hub as a traffic cop

Star topology or network

Data collection utilized by peer review organization to determine the quality of patient care

Uniform Clinical Data Set (UCDS)

Serves as a model for state adoption and provides rules about health information management. As of 1996, only Montana and Washington had enacted this legislation

Uniform Healthcare Information Act (1985)

The patient retains the same number for every encounter at the facility

Unit Numbering

Gives a person access by using his or her name

User-based access

Governing Board has ultimate authority, followed by the CEO, Medical Staff, Department Directors, Supervisors, and Numerous Subordinates

Vertical Organizational Chart

Agreement, valid offer, acceptance, and exchange of considerations

What 3 elements are required for a contract to exist?

A crime is an offense against a person or the public at large. A tort is a civil wrong against an individual

What is the difference between a CRIME and a TORT

an HIM department budget report for may shows a payroll budget of $25,000 and an actual payroll exepnse of $22, 345. the percentage of budget variance for the month is a. $2,655 b. 11% c. $265 d. 0.9%

b. 11%

as a member of the project team for document imaging implementation, you were asked to provide the information in the grid above. this is an example of a a. pert diagram b. gantt chart c. pmbok chart d. work flow diaram

b. gantt chart

your hospital has purchased a number of outpatient facilities. you have been assigned to chair an interdisciplinary committee that will write record retention policies for the new corporation. you begin by telling the committee their primary consideration when making retention decisions must be a. space considerations b. statutory requirements c. provider preferences d. professional standards

b. statutory requirements

American Medical Association (AMA) was established to...?

Assure the quality of American medical education

Made up of a series of fields about one person or thing

Database Record

Based on a specific problem area after the initial baseline review

Focused audit

Requires that all health care facilities notify patients age 18 and over that they have the right to have an advance directive placed in their health record

Patient Self Determination Act (1990)

Operating systems software and application software

Software

SOAP what does S stand for?

Subjective (patient's point of view)

Court order requiring someone to appear in court to give testimony

Subpoena

How do you calculate net autopsy rate?

(Number of inpatients for a given period/Number of deaths of hospital patients whose bodies are available for autopsy) x 100

What are the three elements of privileged communication?

1. Relationship between patient and provider 2. Information must have been acquired through such a relationship 3. Information must have some connection with the provider's task of treating the patient

Promote comparability and compatibility of data by using standard data items with uniform definitions

Minimum Data Set (MDS)

Supervision, room, and board provided for those unable to live independently

Domiciliary

Requires drug and alcohol abuse patient records to be kept confidential and not subject to disclosure except as provided by law

Drug Abuse and Treatment Act (1972)

Legal document in which patients name someone close to them to make decisions about their health care in the event they become incapacitated

Durable Power of Attorney

Private network that has its servers located inside a firewall

Intranet

MINIMUM DATA SET, PROMOTES COMPARABILITY AND COMPATIBILITY OF STANDARD DATA ITEMS USING UNIFORM DEFINITIONS.

MDS

Costs are controlled by the managed-care plan's management of members' uses of services; providers are reimbursed by fee schedules

Managed Fee-For-Service

OUTCOME AND ASSESSMENT INFORMATION SET, STANDARDIZED REPRODUCIBLE ASSESSMENT INSTRUMENT USED TO MONITOR HOME HEALTH CARE.

OASIS

which of the following is the unique identifier in the database illustrated in the table for question 12 a. record number b. patient's last name c. date of birth d. date of service

a. record number

your facility is storing scanned records for long-term storage on optical disk. the risk management committee's disaster task force has recommended that copies of the disks be stored at a facility across town. the administrator is concerned that records may be altered on the disks store off-site. you tell the administrator a. this is a legitimate concern; it should be addressed in the contract written with the storage facility b. this is not a concern because WORM technology makes it impossible to alter the documents c. this is a legitimate concern; perhaps the committee should consider storing duplicates in two locations in this facility d. this is not a concern; there is really no need to make and store duplicate disks as they are difficult to damage.

b. this is not a concern because WORM technology makes it impossible to alter the documents.

the most sophisticated level of interoperability of an EHR and other such systems is the _____ level. a. basic b. functional c. semantic d. exchange

c. semantic

Persons, locations, things, or concepts about which data can be collected and stored

Database Entities

Made up of several characters such as name, age, or gender

Database Field

How long does AHIMA recommend keeping an adult patient health record?

10 years after most recent encounter

Made up of fields and records about an entity such as a patient

Database File

Redefining the workplace with consideration for comfort and safety

Ergonomics

AGENCY FOR HEALTHCARE RESEARCH AND QUALITY, AGENCY, WHOSE FOCUS IS TO PRODUCE AND DISSEMINATE INFORMATION WHICH IMPROVES QUALITY, REDUCES COST, AND ENHANCES THE EFFECTIVENESS OF HEALTH CARE.

AHRQ

AMERICAN MEDICAL INFORMATICS ASSOCIATION, PROVIDES LEADERSHIP IN THE DEVELOPMENT/IMPLEMENTATION OF INFORMATION SYSTEMS TO IMPROVE PATIENT CARE.

AMIA

AMERICAN NATIONAL STANDARDS INSTITUTE. OVERSEES AND ACCREDITS PRIVATE STANDARDS DEVELOPING ORGANIZATIONS IN THE US.

ANSI

AMERICAN SOCIETY FOR TESTING AND MATERIALS, NATIONAL STANDARDS DEVELOPMENT BODY.

ASTM

Starts with last names and then includes first name and middle initial

Alphabetic Filing

What did the Consolidated Omnibus Budget Reconciliation Act (COBRA) establish?

Anti-dumping statue, established criteria for the transfer and discharge of Medicare and Medicaid patients

The simplest network topology that connects one device to another along a "backbone"

Bus or line topology

A character such as a number, letter, or symbol

Byte

Multi-disciplinary standards that outline the processes of care and expected outcomes within predetermined time frames

Care Maps

Records filed in one location

Centralized Filing

The charges themselves plus the underlying cost of services provided are maintained in this database

Charge Description Master (CDM)

Treatment recommendations often based on guidelines, step-by-step description of an accepted procedure

Clinical Protocols

Listing or arrangement of data in a designated order; contains special types of information, and purpose is to assist in location of desired information

Index

Data values should be just large enough to support the application or process and acceptable values or ranges must be defined

Data Precision

DATA SETS ARE DEFINED AS A GROUP OF DATA ITEMS/ELEMENTS AND THEIR DEFINITIONS. EACH ELEMENT IN THE DATA SET COULD ALSO HAVE DEFINED VALUES. WHEN THE VALUE OF THE DATA SET IS DEFINED DATA COLLECTION AND MEASUREMENT OF THE DATA CAN OCCUR.

DATA SETS

UNITED STATES DEPARTMENT OF HEALTH AND HUMAN SERVICES, BRANCH OF THE FEDERAL GOVERNMENT WHO HAS RESPONSIBILITY FOR REGULATORY PROGRAMS AFFECTING THE HEALTH CARE INDUSTRY.

DHHS

Determined the hospital has a duty to provide adequately trained medical staff to patients

Darling vs. Charleston case

Dates, numbers, images, symbols, letters, and words that represent basic facts and observations about people, processes, measurements, and conditions

Data

1. Collection of elements on a given subject 2. Raw facts and figures expressed in text, numbers, symbols, and images 3. Facts, ideas, or concepts that can be captured, communicated, and processed, either manually or electronically

Data (3 definition points)

Data items are easily obtainable and legal to collect

Data Accessibility

Data are the correct values and are valid

Data Accuracy

All required data items included AND entire scope of data is collected and intentional limitations documented

Data Comprehensiveness

Value of data is reliable and consistent across applications

Data Consistency

Data is up to date, if it is outdated it must have been up to date at the time it was presented

Data Currency

Describes an entity or distinct characteristics about it

Database Attribute

Advantage of Integrated Health Record?

Easy to follow course of diagnosis and treatment

Monitors patients who enter the emergency room for services

Emergency Room Register

Health care plan compensates providers with a lump-sum payment to compensate them for all services delivered to a patient for a specific illness and over a specific period of time

Episode of Care

A performance or quality review conducted by a third party payer or consultant hired for the purpose

External audit

How long does AHIMA recommend keeping a minor patient health record?

From the age of majority (usually 18) plus a statute of limitation governing medical malpractice lawsuits

Simplified administration of health insurance and addressed electronic data interchange

HIPPA simplifcation administrative

What is the Department of Health and Human Services (DHHS) responsible for?

Health issues, including health care and cost, welfare of various populations, occupational safety, and income security plans

Data is organized in tree's according to relationships (one to many)

Hierarchical Database Model

Individuals at the top have authority and it passes downward through a chain of command

Hierarchical Organizational Chart

Legislation enacted in 1946 that provided funding for the construction of hospitals and other health care facilities

Hill Burton Act

Selects medical staff members

Hospital's governing board

(Number of C-sections performed/Number of women who delivered) x 100

How do you calculate C-section rate?

(Number of intermediate and late deaths/Number of live births + numerator) x 100

How do you calculate fetal death rate?

Total number of hours worked/Number of full-time hours per week

How do you calculate full time equivalents (FTEs)?

(Number of shelving inches x number of shelves) / inches of an individual medical record

How do you calculate shelving units?

1. Data that have been processed into meaningful form, manually or by computer in order to be valuable to user 2. Adds to a representation and tells recipient something that was not known before

Information (2 definition points)

THE JOINT COMMISSION ON THE ACCREDITATION OF HEALTHCARE ORGANIZATIONS, PRIVATE ACCREDITING ORGANIZATION WHOSE GOAL IS TO IMPROVE THE QUALITY OF PATIENT CARE THROUGH THE DEVELOPMENT/ASSESSMENT STANDARDS OF PERFORMANCE BY HEALTH CARE ORGANIZATIONS.

JCAHO

Supports general managers who focus on managing people and processes as opposed to strategy and structure, horizontal scheme embraces individual capabilities, employees may have two or more supervisors

Matrix Organizational Chart

Series of steps required to move from one situation to another; considered a strategic plan

Migration plan

Comprehensive functional assessment of long-term care patients

Minimum Data Set (MDS) for Long-Term Care (LTC) and Resident Assessment Instrument (RAI)

Health care services are transported to the patients, especially diagnostic procedures

Mobile Diagnostic Services

Nursing care and related services for residents who need medical, nursing, or rehabilitative care provided on a 24 hour basis

Nursing Long-Term Care

Uses commands that act as small, self-contained instructional units (objects) that may be combined in various ways

Object Oriented Database Model

SOAP what does O stand for?

Objective (what the practitioner finds)

Created the Nursing Home Reform Act, which ensures residents of nursing homes receive quality care, requires provision of certain services, and establishes a residents' bill of rights

Omnibus Budget Reconciliation Act (1987)

Hardware, software, transmission, media, and database industry standards allow different computer vendor systems to communicate to each other

Open System Architecture

Maintained for 10 years; provides statistical data for caseload analysis and administrative reports

Operating Room Register

Comprehensive assessment for adult home care patient and forms the basis for measuring patient outcomes

Outcome and Assessment Information Set (OASIS)

Provides every physician with a list of identifying medical cases

Physician Index

SOAP what does P stand for?

Plan (approach to be taken to resolve patient's problem

The graph has the tail off to the right

Positively skewed

Gives individuals some control over the information collected about them by the federal government

Privacy Act (1974)

Recognition and enforcement of the rights and duties of private individuals and organizations

Private Law (Civil Law)

A record used for selected data elements to aid in research conducted by clinical and non-clinical people

Secondary Patient Record

A new number is assigned to the patient for each new encounter at the facility

Serial Numbering

Every unit has the same chance of being included in the sample

Simple random sample

Statutory law written or enacted by bodies such as the U.S. Congress and state and local legislatures

Statutes

Filing charts in sequential order; the records start with the chart with the lowest number value and end with the chart with the highest number value

Straight Numeric Filing

Court order that requires a person to appear in court to testify

Subpoena Ad Testificandum

A written order commanding a person to appear, give testimony, and bring all documents(records) described in the subpoena to court

Subpoena Duces Tecum

Introduced the Peer Review Organization (PRO) program as a component of Medicare law to ensure the quality of care rendered to patients

Tax Equity and Fiscal Responsibility Act (1936) TEFRA

Who owns the physical health record?

The healthcare provider, physician, or hospital that maintains it

UNIFORM AMBULATORY CARE DATA SET, WORKS TO IMPROVE DATA COLLECTED IN THE AMBULATORY SETTING.

UACDS

UNIFORM HOSPITAL DISCHARGE DATA SET- USED BY FEDERAL AND STATE AGENCIES TO COLLECT DATA ON INPATIENTS.

UHDDS

Improve ability to compare data in ambulatory care settings

Uniform Ambulatory Core Data Set (UACDS)

Stipulates that records can be admitted as evidence in a court of law if they were kept in the ordinary course of business.

Uniform Business Records as Evidence Act (1936)

Facilitate charge capture, enables the facility to capture and record patient charges, and allows the provider to accurately charge routine services and supplies to patients

What is the purpose of CDM?

10 years past the age of majority

When does AHA recommend record retention for a minor?

When should H&P be documented in record?

Within 24 hours of admission

What can happen if you disregard a subpoena?

You can be held in contempt of court

the coder works 7.5 hours per day. if a time standard is determined from sample observations to be 2.50 minutes per record for coding emergency room records, what is the daily standard for the number of records coded when a 15% fatigue factor is allowed? a. 153 records per day b. 180 records per day c. 192 records per day d. 200 records per day

a. 153 records per day

when checking the census data at south beach women's center, you see that just yesterday, there were four sets of triplets, five sets of twins, and eight single births. yesterday, south beach women's center had a. 17 deliveries b. 25 deliveries c. 30 deliveries d. 39 deliveries

a. 17 deliveries

the performance standard for coders is 28-33 workload units per day. workload units are calculated as follows: inpatient record=1 workload unit outpatient surgical procedure records= 0.75 workload units outpatient observation/emergency records=0.50 workload units one week's productivity information is shown in in the table above. what percentage of the coders is meeting the productivity standards? a. 100% b. 75% c. 50% d. 25%

a. 100%

based on the information below, what was the net death rate at Seaside Hospital in January? Admission- 280 discharged to home-212 discharge transfers-28 death less than 48 hours- 8 death greater than 48 hours- 6 a. 2.4% b. 2.8% c. 3.8% d.5.8%

a. 2.4%

the correspondence section of your department receives an average of 50 requests per day for release of information. it takes an average of 30 minutes to fulfill each request. using 6.5 productive hours per day as your standard, calculate the staffing needs for the correspondence. a. 3.8 FTE b. 2.5 FTE c. 3 FTE d. 4 FTE

a. 3.8 FTE

in your job as chief security office, you are evaluating software programs taht will support your policy on sound terminal controls within your facility. one of the features you include in your request for information to vendors is a. auto log off b. encryption c. voice recognition feature d. unique identifier for log-on

a. auto log off

reference checks are conducted on potential employees to help assess the applicant's fit with the position and also to a. confirm the accuracy of information provided on the application b. uncover skills the applicant may have be neglected to report c. get another opinion on the applicant's emotional stability d. alert the past employer that the applicant is job hunting

a. confirm the accuracy of information provided on the application

a technique that uncovers new information from existing information by probing data sets is known as a. data mining b. system query language c. neural network analysis d. data warehousing

a. data mining

a supervisor reviews a job to determine the required content, skills, knowledge, abilities, and responsibilities for the position. the tasks are grouped and lines of responsibility and authority are defined. the supervisor is writing a job. a. description b. analysis c. process d. detail

a. description

you are the director of coding and billing at a large group practice. the practice manager stops by your office on his way to a planning meeting to ask about the timeline for complying with HITECH requirements to adopt meaningful use EHR technology. you reply that the incentives began in 2011 and end in 2014. you remind him that by 2015, sanctions for noncompliance will appear in the form of a. downward adjustments to medicare reimbursement b. withdrawal of permission to treat medicare and medicaid patients c. a mandatory action plan for implementing a meaningul use EHR d. monetary fines up to $100,000

a. downward adjustments to medicare reimbursement

your hospital takes advantage of the 8/80 exemption for health care facilities. assuming that no employee worked more than 8 hours in a day, which of the employees listed in the table below will be paid overtime this pay period? a. employees 101 and 105 b. employees 101, 102, and 105 c. employees 101, 104, and 105

a. employees 101 and 105

as the project manager for the upcoming EHR implementation, you ask your assistant to develop a work breakdown structure (WBS). critical to implementations and project success, the WBS a. lists steps needed to complete the project b. determines dependencies among project tasks c. describes project responsibilities d. define the project's critical path

a. lists steps needed to complete the project

a P value of less than 0.05 is what researches commonly use to reject the null hypothesis. a smaller P value may place interpretation of the results of the study at risk fora a. sampling error b. stratification error c. type 1 (a) error d. type 2 (b) error

a. sampling error

which of the following employees would be considered exempt under the fair labor standards act a. the head of the department of health information services who is involved in decision making and planning 90% of the time b. the coding supervisor who has responsibility for three employees and performs analysis and coding 80% of the time c. the departmental secretary who is responsible for performing a variety of clerical and administrative tasks d. the sole employee in the physicians workroom who has responsibility for maintaining and tracking medical record deficiencies

a. the head of the department of health information services who is involved in decision making and planning 90% of the time

a clerical-level employee reports an incident in which the clerk felt the first-line supervisor discriminated on the basis of the clerk's gender. the best action for you to take at this time is to a. thoroughly investigate the matter and document your findings b. talk with the first-line supervisor to determine what happened. c. ask the other clerical level staff if they have had similar experiences d. ask the clerk to provide objective evidence of the discrimination

a. thoroughly investigate the matter and document your findings

Each month the staff of the clinic with the lowest overall waiting time is awarded a free dessert in the Gulfside healthcare center cafeteria. take a look at the information listed above. the winner will be selected based on a. demonstrative clinical data b. comparative aggregate data c. objective individual data d. duplicate thematic data

b. comparative aggregate data

Patient was seen in the Emergency Department with lacerations on the left arm. Two lacerations, one 7 cm and one 9 cm, were closed with layered sutures 12002 simple repair of superficial wounds of scalp, neck, axillae, external genitalia, trunk and/or extremities(including hands and feet); 2.6-7.5cm 12004 simple repair of superficial wounds of scalp, neck, axillae, external genitalia, trunk and/or extremities (including hands and feet); 7.6-12.5 cm 12035 Layer closure of wounds of scalp, axillae, trunk and/or extremities (excluding hands and feet) 12.6-20.0cm 12045 Layer closure of wounds of neck, hands, feet and/or external genitalia; 12-6-20.0 cm a. 12045 b. 12035 c. 12002, 12004 d. 12004

b. 12035

based on the MS-DRG report above, what is the case-mix index for this facility? a. 0.204193 b. 2.965807 c. 11.639 d. 57

b. 2.965807

the patients family asked the attending physician to keep the patient in the hospital for a few days more until they could make arrangements for the patient's home care. because the patient no longer meets criteria for continued stay, if the physician complies with the family's request, this would be considered a. the best utilization of the hospital's resources. b. an inappropriate use of hospital resources c. a compassionate use of the hospital's resources d. appropriate provided it is limited to a few days

b. an inappropriate use of hospital resources

you have been the supervisor in quality management at Naples hospital for almost a year now and your appraisal is right around the corner. your director has asked the members of your staff and the physicians on the committees you service to complete forms that will provide input into the appraisal process. in addition, you have been asked to formally assess your own progress in this new position. it is time-consuming and more than a little intimidating, but you suppose it will provide you with some good insights into the quality of your work. because you are, after all, the supervisor in QM, it is only fair that you participate fully in your facility's commitment to a. participative evaluation b. 360-degree evaluation c. conductive evaluation d. managed evaluation

b. 360-degree evaluation

a major drug company wants to promote a fundraiser targeting patients with congestive heart failure. the drug company representative has requested a list of patients treated at your facility. as privacy and security officer, you tell them that a. they just need to send a written request for the list b. a prior authorization is required before any PHI can be released c. you will need to confer with the medical director d. if the fundraising was conducted by a business associate without authorization, and the funds were to benefit your facility (the covered entity), that you could disclose the information

b. a prior authorization is required before any PHI can be released

the cfo of your facility asks you to prepare a budget for the fiscal year based on the past volume and expected capacity for the coming year. this process is an example of using the "___________" budgeting method a. rolling budget b. fixed budget c. flexible or statistics budget d. zero-based budget

b. fixed budget

the committee that is preparing your acute care hospital for an electronic health record is planning for an imaging system for record archiving in the immediate future. they are looking for a solution for data interfacing or integration of the imaging system into other computer systems. you recommend a. data dictionary guidelines b. health level 7 standards c. regional health information organization guidelines d. joint commission standards

b. health level 7 standards

use the information provided in the table above to calculate the delinquent rate. the delinquent rate a. cannot be determined b. is 36% c. is 50% d. is 71%

b. is 36%

ORYX is a program that was developed by a. CMS to track Medicare costs b. joint commission to link patient outcomes to accreditation c. NIH to track communicable disease d. AMA to allow for rapid CPT updates

b. joint commission to link patient outcomes to accreditation

a tool that is used to illustrate the various applications, support structures, and sequencing of implementation phases for a system is a a. work flow diagram b. migration path c. milestone chart. d. pertt diagram

b. migration path

you have been asked to reduce your department's operating budget by 20%. in order to do so, you will have to effect reductions in your largest budget line. you will have to make cuts in a. equipment b. personnel c. supplies d. contracts

b. personnel

everyone in the health information department has been working overtime to complete a major record conversion. the supervisor will have to plan for overtime pay for all personnel who are not a. hourly employees b. salaried exempt employees c. salaried nonexempt employees d. temporary employees

b. salaried exempt employees

a common goal of the office of the national coordinator for health information technology, RHIOs, and a national infrastructure for information is a. translating images into a digital format b. sharing information among providers c. transferring health information within a hospital system d. promoting telemedicine

b. sharing information among providers

diverticulitis large bowel with abscess; right hemicolectomy with colostomy performed K57.90 Diverticulosis of colon (without mention of hemorrhage) K57.92 Diverticulitis of colon (without mention of hemorrhage) K63.0 Abscess of intestine ODTK0ZZ Open and other right hemicolectomy OKTL0ZZ Open and other resection of transverse colon OD1K0Z4 Exteriorization of large intestine (loop colostomy) a. K57.90, ODTL0ZZ, OD1K0Z4 b. K57.92, ODTK0ZZ, OD1K0Z3 c. K57.92, K63.0, ODTK0ZZ d. K57.92, K63.0, ODTL0ZZ

c. K57.92, K63.0, ODTK0ZZ

Key west hospital collected the data displaye above concerning its four highest volume MS-DRGs. which MS-DRG generated the most revenue for the hospital? a. MS-DRG A b. MS-DRG B c. MS-DRG C d. MS-DRG D

c. MS-DRG C

the MS-DRG weight in a particular case is 2,0671 and the hospital's payment rate is $3,027. how much would the hospital receive as reimbursement in this case? a. $3,027.00 b. $5,094.10 c. $6, 257.11 d. $960.00

c. $6,257.11

you supervise five clerical employees who will be moving when a new wing of your facility is completed. when you meet with the architect to plan their space, you will ask for a. 200 square feet of space for you clerical staff b. 250 square feet of space for your clerical staff c. 300 square feet of space for your clerical staff d. 350 square feet of space for your clerical staff

c. 300 square feet for your clerical staff

there are 15 employees in your department. there were 21 working days last month. there were a total of 12 lost workdays last month due to absenteeism of all types. the absenteeism rate for your department last month was a. 21% b. 57% c. 4% d. 5%

c. 4%

a patient who was admitted to the hospital on January 14 and discharged on March 2 in a nonleap year has a length of stay of a. 45 days b. 46 days c. 47 days d, 48 days

c. 47 days

community hospital administration decided to change the number of adult and children beds from 300 to 375 effective July 1. the total number of inpatient services days for adults and children for the year was 111, 963. what was the percentage of occupancy rate for adults and children for the entire year? a. 0.9% b. 45.4% c. 90.8% d. 91.0%

c. 90.8%

your facility has decided to purchase an integrated patient information system. your part in the initial work plan is to develop system specifications that will ultimately be sent out to vendors who will potentially submit a bid on your system. you are working on the systems specs that will become part of the a. CPR b. IRB c. RFP d. CRS

c. RFP

which of the following statements is true a. a surgical procedure may include one or more surgical operation b. the terms surgical operation and surgical procedure are synonymous c. a surgical operation may include one or more surgical procedures d. the term surgical procedure is an incorrect term and should not be used.

c. a surgical operation may include one or more surgical procedures

in order to perform their jobs, facility employees should have full and timely access only to the information they need to complete the task at hand. this is similiar to HIPAA's provision for a. a notice of privacy practices b. amending a record c. accessing need to know information only d. an informed consent

c. accessing need to know information only

as supervisor of the cancer registry, you report the registry's annual caseload to administration. the most efficient way to retrieve this information would be to use a. patient abstracts b. patient index. c. accession register d. follow-up files

c. accession register

stage I of meaningful use focuses on data capture and sharing. which of the following is included in the menu set of objectives for eligible hospitals in this stage a. use CPOE for medication orders b. smoking cessation counseling for MI patients c. appropriate use of HL-7 standards d. establish critical pathways for complex, high-dollar cases

c. appropriate use of HL-7 standards

your facility has a team that has been working to develop a strong performance improvement model, and they have come up with the model show above. the team asks if you see anything missing from the model. you tell them they. a. are missing a step requiring regular employee input into the process. b. are missing a step requiring reporting to the board of directors c. are missing a step requiring ongoing monitoring and reassessment d. aren't missing any steps, the model is a good one

c. are missing a step requiring ongoing monitoring and reassessment

the emergency department staff has complained that the clerical staff in your department is delaying stat reports. you decided to meet with your staff and develop a cause and effect diagram to determine possible reasons for the delay. you have explained the issue to your staff and have set up a blank cause and effect diagram. the next step is to a. discuss the importance of prompt delivery of stat reports b. determine whether there are internal conflicts in the area c. brainstorm possible reasons for delays in delivering the reports d. design a new system that will support prompt report delivery

c. brainstorm possible reasons for delays in delivering the reports

four people were seen in your emergency department yesterday. which one will be coded as a poisoning? - josh was diagnosed with digitalis intoxication -ben had an allergic reaction to a dye adminstered for a pyelogram - bryan developed syncope after taking Contac pills with a double scotch -matthew had an idiosyncratic reaction between two properly administered prescription drugs a. josh b. ben c. bryan d. matthew

c. bryan

the decision makers in the HIM department have decided to use the decision analysis matrix method to select coding software. use of this method will help ensure a. all alternatives/vendors are evaluated subjectively b. the personalities of individual vendors will not influence the decision c. consistent criteria are used to evaluate the alternatives/vendors d. the level of software support will be considered in the decision

c. consistent criteria are used to evaluate the alternatives/vendors

in order to prevent the accidental introduction of a virus into your facility's local area network, your facility has a policy that strictly prohibits a. doing personal work on the computer system, even during personal time. b. sharing disks from one workstation to another within the facility c. downloading executable files from electronic bulletin boards d. sending or receiving e-mail from addresses that have not be authorized

c. downloading executable files from electronic bulletin boards

the use of "public" and "private" keys is part of what type of data protection a. firewalls b. biometrics c. encryption d. phishing

c. encryption

many of the departments in your facility create and modify forms often. a major key to forms control in this setting is a. consistent formatting of each page of each form b. capturing every data item required by UHDDS c. giving each form or view an identifiable name, number, and revision date. d. providing instructions when necessary for appropriate data fields

c. giving each form or view an identifiable name, number, and revision date.

part of your job description is to educate physicians regarding proper documentation policies and standards. you are the a. information security manager b. clinical data specialist c. health information manager d. risk manager

c. health information manager

take a look at the comparison of the two life cycles below. look at the cell in the information systems life cycle that is filled with question marks. which of the following should replace those question marks to make a complete, correct comparison a. growth b. installation c. implementation d. reevaluation

c. implementation

you are conducting an educational session on benchmarking. you tell your audience that the key to benchmarking is to use the comparison to a. implement your QI process b. make recommendations for improvement c. improve your department's processes d. compare your department with another

c. improve your department's processes

encoding software is installed at your hospital 2 years ago. the coders are well trained on it and like using it. it functions well and only requires ICD code updates yearly. in terms of the information system life cycle phases, the coding system is likely in a. design phase b. implementation phase c. operation and maintenance phase d. obsolescence or decline phase

c. operation and maintenance phase

patients with diabetes participated in a study to determine the effectiveness of a new drug. the drug was taken at bedtime. the drug company expected patients taking glucodown to have a normal early morning fasting blood sugar level. the null hypothesis for the study follows there will be no difference in fasting early morning blood sugar levels between patients taking glucodown and patients take a placebo. half the patients were given glucodown. half were given a placebo. early morning fasting blood sugar levels are reported below. the researches in the previous question wonder if there is any relationship between patient age and average fasting blood sugar. the best data display tool the researchers could use to look for a possible relationship would be a a. pareto diagram b. line graph c. scatter diagram d. cause and effect diagram

c. scatter diagram

as a new HIM manager, you recognize that employee development is a necessary investment for the long-term survival and growth of the organization. your goal is to design and implement a staff development program for your employees, so one of you first steps it to a. implement training programs that emphasize teamwork b. establish a budget for all hospital employee training c. survey the HIM employees to assess their need for new skills or knowledge d. establish HIPAA training programs hospital-wide

c. survey the HIM employees to assess their need for new skills or knowledge

the difference between an institutional review board (IRB) and a hospital's ethics committee is that a. the IRB focuses on patient care only, and the ethics committee addresses both patient care and business practices b. the ethics committee reviews ethics complaints, and the IRB focuses on developing policies and procedures c. the IRB deals with the ethical treatment of human research subjects, and the ethics committee covers a wide range of issues d. the IRB is made up entirely of patient care providers, and the ethics committee is mulitdisciplinary

c. the IRB deals with the ethical treatment of human research subjects, and the ethics committee covers a wide range of issues

in reviewing the policies on release of information in respect to the privacy rules, you not that it is still acceptable to allow release of protected health information without patient permission to a. the patient's spouse b. a health care provider interested in the case c. the quality assurance committee for review purposes d. a third-party payer with a direct interest in the case

c. the quality assurance committee for review purposes

as the HIM manager in charge of your department's budget, you are mandated to report on variances of more than 6% either positive or negative to your chief financial officer and include the reasons for the variance and any action plans necessary. based on the table below for the december variance report, what category would you be required to report on the CFO? a. office supplies b. contract services c. travel d. education

c. travel

in the past, joint commission standards have focused on promoting the use of facility approved abbreviation list to be used by hospitals care providers. with the advent of the commission's national patient safety goals, the focus has shifted to the a. prohibited use of any abbreviations b. flagrant use of specialty-specific abbreviations c. use of prohibited or "dangerous" abbreviations d. use of abbreviations used in the final diagnoses

c. use of prohibited or "dangerous" abbreviations

evidence-based management and decision making is an emerging model now used to make more informed decisions. the premise of this model is a. using intuition based on previous experience. b. using a decision tree with branches that show the alternatives c. using the best clinical and research practices available d. using an alternative that meets minimum requirements

c. using the best clinical and research practices available

a transcription unit has been asked to tally the number of times they have to leave sections of a report blank for various reasons (poor dictation technique, background noise, etc.) the quality improvement tool most likely to help collect these data would be a. force field analysis b. decision matrix c. flowchart d. check sheet

d. check sheet

annual costs for the only release of information clerk at jacksonville beach healthcare center (salary and benefits) are $36,429. the monthly cost for the copier used solely for ROI is $89 (supplies and repair). it costs the department $0.95 on average for ROI mailings (envelopes and postage). there were 687 requests filed for ROI last month. The cost per request for release of information last month was. a. $4.42 b. $4.55 c. $4.63 d. $5.50

d. $5.50

Patient was seen for excision of two interdigital neuromas from the left foot 28080 excision, interdigital(morton) neuroma, single, each 64774 excision of neuroma; cutaneous nerve, surgically identifiable 64776 excision of neuroma; digital nerve, one or both, same digit a. 64774 b. 64776 c. 28080 d. 28080, 28080

d. 28080, 28080

how long will it take to complete the project described below? a. 12 days b. 17 days c. 20 days d. 29 days

d. 29 days

now that the EHR has been full implemented, your are ready to move old records to basement storage. you are ordering shelving for those old paper files. you have 18,000 records. the files average is three files per filing inch. the shelf units you have selected have six shelves that will hold 34 inches per shelf. you will have to plan for a 20% expansion rate to accommodate miscellaneous paper records over the next 10 years. how many shelving units should you order? a. 30 b. 31 c. 35 d. 36

d. 36

a 4-year-old had a repair of an incarcerated inguinal hernia. this is the first time this child had been treated for this condition 49496- repair initial inguinal hernia full-term infant, under age 6 months, or preterm infant over 50 weeks' post 49501- repair initial inguinal hernia, age 6 months to under 5 years, with or without hydrocelectomy; incarcerated or strangulated 49521- repair recurrent inguinal hernia, any age; incarcerated or strangulated 49553- repair initial femoral hernia, any age; incarcerated or strangulated a. 49553 b. 49496 c. 49521 d. 49501

d. 49501

a quantitative drug assay was performed for a patient to determine digoxin level 80050- general health panel 80101- drug screen, qualtitative; single drug class method (e.g. immunoassay, enzyme assay), each drug class 80162- digoxin (therapeutic drug assay, quantitative examination) 80166- doxepin (therapeutic drug assay, quantitative examination) a. 80101 b. 80050 c. 80166 d. 80162

d. 80162

the file clerks in your department's main file area report that they are able to locate 400 out of 450 requested records during the past month. there are a total of 4,500 records in the main file. what is the area's accuracy rate? a. 1.1% b. 8.9% c. 10.0% d. 88.9%

d. 88.9%

which of the following scenarios identifies a pathologic fracture? a. greenstick fracture secondary to fall from bed b. compression fracture of the skull after being hit with a baseball bat c. vertebral fracture with cord compression following a car accident d. compression fracture of the vertebrae as a result of bone metastasis.

d. compression fracture of the vertebrae as a result of bone metastasis.

a ______ uses a private tunnel through the internet as a transport medium between locations for secure access and transmission a. TCPIP protocol b. firewall c. hub. d. VPN

d. VPN

postage charges in the Health information department have increased during the last quarter. the department director has seen metered envelopes in the mail bin that do no appear to be those used for departmental business. the best course of action for the director would be to a. remove the postage meter from the department b. keep a watchful eye on the meter and who uses it c. issue employee warnings at the next departmental meeting d. assign responsibility for the postage meter to one employee

d. assign responsibility for the postage meter to one employee

the purpose of the correct coding initiative is to a. increase fines and penalties for bundling services into comprehensive CPT codes b. restrict medicare reimbursement to hospitals for ancillary services c. teach coders how to unbundle codes d. detect and prevent payment for improperly coded services

d. detect and prevent payment for improperly coded services

an effective means of protecting the security of computerized health information would be to a. require all facility employees to change their passwords at least once a month b. write detailed procedures for the entry of data into the computerized information system c. install a system that would require fingerprint scanning and recognition for data access. d. develop clear policies on data security that are supported by the top management of the facility

d. develop clear policies on data security that are supported by the top management of the facility

your facility would like to improve physician documentation in order to allow improved coding. as coding supervisor, you have found it very effective to provide the physicians with a. a copy of the facility coding guidelines, along with written information on improved documentation b. the UHDDS and information on where each data element is collected and/or verified in your facility c. regular in-service presentations on documentation, including its importance and tips for improvement d. feedback on specific instances when improved documentation would improve coding

d. feedback on specific instances when improved documentation would improve coding

your HIM department is moving to a new location and in order to arrange your employees and functions for optimal work flow efficiency and to decide which employees need to be placed close to each other, the tool you decide to use is a a. data flow diagram b. PERT chart c. proximity chart or movement diagram d. flow process chart.

d. flow process chart

the percentage of records delinquent due to the absence of an operative report a. is 1.7% b. is 2.7% c. is 4.6 % d. cannot be determined from the information given

d. is 4.6%

you are a new supervisor in HIM department and find it difficult to deal with performance issues. Laney, an employee in the release of information section, hs been late several times this month. she has already been given a verbal warning. she was late again today. according to the progressive discipline process your next step will be to a. reinforce the institution's policies b. reissue the verbal warning c. suspend the employee d. issue a written warning.

d. issue a written warning.

you stop by the office to meet a friend for lunch. looking on her desk, you see the grid above. your friend is trying to. a. plan a conversion b. design a system c. analyze a workflow d. make a decision

d. make a decision

a portion of a deficiency slip is reproduced below. this patient was discharged yesterday. your greatest concern regarding deficiencies on this record would be the missing. a. signature on the physical exam b. signature on the discharge summary c. diagnosis and procedures d. operative report

d. operative report

a clerk's work performance has diminished dramatically during the past 2 weeks. the supervisor initiates a discussion with the clerk, during which the clerk reveals that he recently accepted that he has an alcohol addiction. the clerk stats an intention to quit drinking completely. the supervisor should. a. terminate the clerk if it can be proved that alcohol was used on the job b. suspend the clerk if alcohol has diminished the clerk's job performance c. give the clerk a leave of absence until these problems can be resolved d. refer the clerk to the facility's employee assistance program

d. refer the clerk to the facility's employee assistance program

which means of data modeling is illustrated in the table shown above? a. entity-relationship model b. object-oriented model c. data management model d. relational data model

d. relational data model

a section of a job description states that the incumbent will handle day-to-day operations in the transcription and release of information areas. this section defines the a. skills required to perform the job. b. time required for each function c. authority associated with the job d. scope of responsibility in the job.

d. scope of responsibility in the job.

a piece of objective data collected upon initial assessment of the patient is a. review of systems b. history of present illness c. chief complaint d. vital signs

d. vital signs

in conducting an educational session for your staff about implementing a benchmarking program, you tell your staff that when an organization uses benchmarking, it is important to compare your facility's outcomes to a. nationally known facilities b. larger facilities c. facilities within your corporation d. facilities with superior performance

facilities with superior performance

The physician has documented the final diagnoses as acute myocardial infaraction, COPD, CHF, hypertension, atrial fibrillation, and status-post cholecystectomy. The following conditions should be reported: I10 Essential Hypertension, benign I10 Essential Hypertension, unspecified I11 Hypertension, heart disease, unspecified as to malignant or benign, with heart failure I21.3 Acute Myocardial infarction, unspecified site, initial episode of care I48.91 Atrial Fibrilation I50.9 Congestive Heart Failure, unspecified J44.9 Chronic Obstructive Pulmonary Disease Z90.89 Acquire absence of gallbladder a. I21.3, J44.9, I21.3, I48.91, Z90.89 b. I11, J44.9, I50.9, I10, I48.91 c. I11, J44.9, I50.9, I10, I48.91, Z90.89 d. I11, J44.9, I50.9, I10, I48.91

B. I11, J44.9, I50.9, I10, I48.91

a patient is admitted through the emergency department. three days after admission, the physician documents uncontrolled diabetes mellitus. what is the "present on admission" (POA) indicator for uncontrolled diabetes mellitus? a. "Y" b. "U" c. "W" d. "N"

D. "N"

The patient had a thrombectomy, without catheter, of the peroneal artery, by leg incision. 34203 embolectomy or thrombectomy, with or without catheter; popliteal-tibioperoneal 35226 repair blood vessel, direct; lower extremity 35302 thromboendarterectomy, including patch graft if performed; superficial femoral artery 37799 unlisted procedure, vascular surgery a. 34203 b. 37799 c. 35302 d. 35226

a. 34203

Office visit for 43-year old male, new patient, with no complications. patient is applying for life insurance and requests a physical examination. a detailed health and family history was obtained and a basic physical was done. physician completed life insurance physical form at patient's request. blood and urine were collected 99381- initial comprehensive preventive medicine evaluation and management of an individual including an age and gender appropriate history, examination, counseling/ anticipatory guidance/risk factor reduction interventions, and the ordering of appropriate immunizations, laboratory/diagnostic procedures, new patient; infant (age under 1 year) 99386-initial comprehensive preventive medicine evaluation and management of an individual including an age and gender appropriate history, examination, counseling/ anticipatory guidance/risk factor reduction interventions, and the ordering of appropriate immunizations, laboratory/diagnostic procedures, new patient; 40-64 years 99396- periodic comprehensive preventive medicine reevaluation and management of an individual including an age and gender appropriate history, examination, counseling/ anticipatory guidance/risk factor reduction interventions, and the ordering of appropriate immunizations, laboratory/diagnostic procedures, established patient; 40-64 years 99450- basic life and/or disability examination that includes completion of a medical history following a life insurance pro forms a. 99450 b. 99386 c. 99396 d. 99381

a. 99450

as the director of a health information technology program, your community college has been selected to participate in the workforce development of electronic health record specialists as outlined by ARRA and HITECH. in order to keep abreast of changes in this program, you will need to regularly access the web site of which governmental agency? a. ONC b. CMS C. OSHA d. CDC

a. ONC

you are implementing a quality improvement plan that utilizes the PDSA cycle. if you correctly implement the PDSA, which pase of the project will take the most of your time a. P b. D c. S d. A

a. P

Parker has type 1 diabetes with hypertension taht is currently controlled with medication. parker was admitted through the ED for an emergency appendectomy. following surgery, the patient developed an infection at the wound site that was treated with antibiotics. when making decision about sequencing the codes for this case, the coder should rely on definitions found in the a. UHDDS b. coding clinic c. CMS coding guidelines d. federal register

a. UHDDS

The discharge diagnosis for this inpatient encounter is rule out myocardial infarction. the coder would assign. a. a code for myocardial infarction b. a code for the patient's symptoms c. a code for an impending myocardial infarction d. no code for this condition

a. a code for myocardial infarction.

the ER staff has collected the data on the number of visits and corresponding wait times in the ER. the data are displayed on the chart shown above. based on this information, what kind of correlation do you see between the number of visits (variable X) and the wait times (variable y)? a. a positive correlation between variable X and variable Y b. a negative correlation between variable x and variable y c. a conjunctive correlation between variable X and variable Y d. a causative correlation between variable x and variable y

a. a positive correlation between variable X and variable Y

as the information security office at your facility, you have been asked to provide examples of technical security safeguards as a result of HIPAA legislation. which of the following would you provide? a. audit controls b. evidence of security awareness training c. surge protectors d. workstation use and location

a. audit controls

which of these conditions are always considered "present on admission" (POA)? a. congenital conditions b. E codes c. acute conditions d. possible, probable, or suspected conditions

a. congenital conditions

sunset beach clinic allows patients to communicate by e-mail to ask questions regarding their treatment and request appointment changes. e-mails and text messages are a. considered health care business records and are subject to the same regulations as records created in face-to-face patient encounters. b. considered proof of patient contact and should be summarized in a progress note in the patient record c. generally maintained in a facility's electronic mail system until the next face-to-face patient encounter d. not typically maintained or documented as patient encounters.

a. considered health care business records and are subject to the same regulations as records created in face-to-face patient encounters

the hospital's strategic plan calls for having the entire health record contact recorded in discrete form within the next 10 years. which system will the HIM director most likely recommend in the early stages of the project as a transition strategy? a. electronic document management system b. clinical data repository system c. CPOE system d. speech recognition system

a. electronic document management system

Johnston city was set upon by a swarm of killer bees. all 5,000 residents are at risk of a bee attack. if 25 residents were attacked by the bees, the incidence of bee attack a. is 5 in 1,000 b. is 5 in 5,000 c. is 25 in 1,000 d. cannot be determined at this time

a. is 5 in 1,000

In reviewing a health record for coding purposes, the coder notes that the patient was put on Keflex post-surgery. there is no mention of a postoperative complication in the attending physician's discharge summary. before querying the doctor, the coder will seek to confirm the infection by reviewing the a. lab report b. nurses' notes c. operative report d. pathology report

a. lab report

sun city repoted 12 cases of chronic heart disease in a population of 8,000 in 2008. in 2011, sun city reported there were still 12 cases of chronic heart disease, but its population had decreased to 6,000. this represents an increase in the a. prevalence of chronic heart disease in sun city b. incidence of chronic heart disease in sun city c. reliability of reporting chronic heart disease in sun city d. occurence of chronic heart disease in sun city.

a. prevalence of chronic heart disease in sun city

release of information has increased its used of part-time prn clerical support in order to respond to increased requests for release of information. the budget variance report will reflect a. the increase in the cost of part-time clerical support for ROI but not the increase in revenue from this area b. the increase in revenue from increased volume in ROI but not the increased costs of part-time clerical support c. both the increases in revenue and increased costs for clerical support in ROI d. neither the increased costs nor increased revenue, as temporary changes are rarely reflected on variance reports

a. the increase in the cost of part-time clerical support for ROI but not the increase in revenue from this area.

a 16-year-old maled was treated at your facility for a closed head injury. the patient's 18-year-old wife accompanied him to the hospital and signed the consent for admission and treatment because of the patient's incapacity at the time. the patient has requested that copies of his medical records be sent to his attorney. who should sign the authorization to release the records? a. the patient b. either of the patient's parents c. the patient's parent or legal guardian d. the patient's wife

a. the patient

in your state, it is legal for minors to seek medical treatment for a sexually transmitted disease without parental consent. when this occurs, who would be expected to authorize the release of the medical information documented in this episode of care to the patient's insurers? a. the patient b. a court-appointed guardian on behalf of the patient c. the custodial parent of the patient d. the patient's doctor on behalf of the patient

a. the patient

If the same condition is described as both acute and chronic and separate sub entries exist in the ICD-10-CM alphabetic index at the same indentation level a. they should both be coded, acute sequence first. b. they should both be coded, chronic sequence first c. only the acute condition should be coded. d. only the chronic condition should be coded

a. they should both be coded, acute sequence first.

Which of the following is coded as an adverse effect in ICD-9-CM? a. tinnitus due to allergic reaction after administration of ear drops b. mental retardation due to intracranial abscess c. rejection of transplanted kidney d. nonfunctioning pacemaker due to defective soldering

a. tinnitus due to allergice reaction after administration of ear drops

a patient with lung cancer and bone metastasis is seen for complex treatment planning by a radiation oncologist 77236- therapeutic radiology treatment planning; complex 77290- therapeutic radiology simulation-aided field setting; complex 77315- teletherapy, isodose plan (whether hand or computer calculated); complex (mantle or inverted Y, tangential ports, the use of wedges, compensators, complex blocking, rotational beam, or special beam considerations) 77334- treatment devices, design and construction; complex(irregular blocks, special shields, compensators, wedges, molds or casts) a. 77315 b. 77263 c. 77290 d. 7734

b. 77263

the proposed national healthcare information network (NHIN) dimensions are graphically depicted in the diagram above. roman numeral are added to allow for the identification of specific areas. where would information concerning a patient's health insurance be located on this chart? a. I b. II c. III d. V

b. II

Patient is admitted for elective cholecystectomy for treatment of chronic cholecystititis with cholelithiasis. prior to administration of general anesthesia, patient suffers cerebral thrombosis. surgery is subsequently canceled. code and sequence the coding from the following codes. I66.9 cerebral thrombosis without cerebral infarction K80.10 chronic cholecystitis with cholelithiasis Z53.09 Iatrogenic cerebrovascular infarction or hemorrhage OFT40ZZ Cholecystectomy, total a. I97.821, K80.10, OFT4OZZ b. K80.10, I66.9, Z53.09 C. I97.821, I66.9, Z53.09 d. I66.9, Z53.09

b. K80.10, I66.9, Z53.09

You are calculating the fee schedule payment amount for physician services covered under Medicare Part B. you already have the relative value unit figure. the only other information you need is a. the facility's case-mix index b. a national conversion factor c. the facility's base rate d. ms-drg relative weights

b. a national conversion factor

you are starting your new job as the sole HIM professional at a small psychiatric practice. the practice uses DSM for billing purposes. you find this "theoretically" reasonable because DSM a. is a widely used and accepted classification system b. codes are also valid ICD-10-CM codes c. codes are also valid CPT codees d. is the industry standard for physchiatric billing systems

b. codes are also valid ICD-10-CM codes

which of the following responsibilites would you expect to find on the job description of a facility's information security officer but NOT on the job description of chief privacy officer? a. cooperate with the office of civil rights in compliance investigations b. conduct audit trails to monitor inappropriate access to system information c. oversee the patient's right to inspect, amend, and restrict access to protected health information d. monitor the facility's business associate agreements

b. conduct audit trails to monitor inappropriate access to system information

a patient has written to request a copy of his own record. when the clerk checked the record, it was noted that the patient was last admitted to the psychiatric unit of the facility. you advise the clerk to a. comply with the request immediately b. contact the patient's attending physician before complying c. ignore the request and advise you if it is repeated d. aske the patient to send the required fee prior to the release

b. contact the patient's attending physician before complying

Make Me Better Clinic (MMBC) provides well child visits and childhood immunizations for four insurance companies. data on the services they provided and the reimbursement they received from the four companies are listed in the two tables below. MMBC receives the best reimbursement for well child visits from a. lifecare b. getwell c. surehealth d. behealthy

b. getwell

Make Me Better Clinic (MMBC) provides well child visits and childhood immunizations for four insurance companies. data on the services they provided and the reimbursement they received from the four companies are listed in the two tables below. Most of the children who are seen at MMBC will have a well child visit and two immunizations. If you add the reimbursement for two immunizations to the reimbursement for each well child visit, which insurance company benefits MMBC most? a. lifecare b. getwell c. surehealth d. behealthy

b. getwell

sally is a HIM professional with many years of experience. unlike some of her colleagues, sally loves the challenge of adapting to change. she is happy that HIPAA empowers the secretary of DHHS to adopt standards for electronically maintained health information. sally hopes that the standardization under HIPAA will make it easier to design safeguards for electronic data, to protect against unauthorized access, to a. make and use copies of the data, and to guard against unauthorized data integration b. protect electronic records from corruption, and to prosecute hackers under federal law. c. prevent the corruption of electronically stored data, and to protect the integrity of the information itself d. submit revisions of claims as they are denied, and to track third-party payers

c. prevent the corruption of electronically stored data, and to protect the integrity of the information itself.

a pharmacist at your facility was caught running a drug ring. the pharmacist filed orders of valuable medications with cheap outdated ones purchased on the internet and then sold the good drugs for profit. patients have been injured and the lawsuits are starting. unfortunately, your facility is going to be held responsible for the pharmacist's negligent acts under the doctrine of a. adjudicus res b. res ipsa loquitur c. respondeat superior d. stare decisis

c. respondeat superior

when a patient is admitted because of a primary neoplasm with metastasis and treatment is directed toward the secondary neoplasm only a. code only the primary neoplasm as the principal diagnosis b. the primary neoplasm is coded as the principal diagnosis, and the secondary neoplasm is coded as an additional diagnosis c. the secondary diagnosis is coded as the principal diagnosis, and the primary neoplasm is coded as an additional diagnoses. d. code only the secondary neoplasm as the principal diagnosis

c. the secondary diagnosis is coded as the principal diagnosis, and the primary neoplasm is coded as an additional diagnosis

in preparing the retention schedule for health records, the most concrete guidance in determining when records may be destroyed will be a. the average readmission rate for the facility b. the available options for inactive records c. the statute of limitations in your state d. joint commission and AOA standards regarding minimum retention periods.

c. the statute of limitations in your state

the chief of staff, chief of medicine, president of the governing body, and most departmental managers have already completed CQI training. unfortunately, the hospital admistrator has not been to training, refuses to get involved with CQI, and refuses to let the administrative departmental staff get training. a. this level of involvement is enough to meet joint commission standards b. the joint commission only expects involvement from clinical staff c. this will not do because it violates joint commission standards and CQI philosophy d. if you can talk him into training his staff, you can let him skip the training

c. this will not do because it violates joint commission standards and CQI philosophy

four HIT students are working on a LOS project at their clinical site. to describe the variability of the data, Smithson suggest using range because it is the most exact. Donaldson wants to use the mean because it is easy to calculate. franklin recommends variance because it is the most preferred and has better interpretive uses. who made the best suggestion a. smithson b. donaldson c. franklin d. howell

d. howell

an established patient was seen by the physician in the office for DTaP vaccine and Hib 90471- immunization administration (includes percutaneous, intradermal, subcutaneous, intramuscular injections); one vaccine (single or combination vaccine/toxoid) 90700-diptheria, tetanus toxoids, and acellular pertussis vaccine DTaP, when admistered to indivduals younger than 7 years, for intramuscular use 90720-diptheria, tetanus toxoids, and whole cell pertussis vaccine and hemophilus influenza B vaccine (DTP-Hib), for intramuscular use. 90721- diptheria, tetanus toxoids, and acellular pertussis vaccine and hemophilus influenza B vaccine (DTaP-Hib), for intramuscular use 90748- Hepatitis B and Hemophilus influenza b vaccine (HepB-Hib), for intramuscular use 99211- office or other outpatient visit for the evaluation and management of an established patient, which may not require the presence of a physician. usually, the presenting problems are minimal. typically 5 minutes are spent performing or supervising procedure a. 90721 b. 90720, 90471 c. 90700, 90748, 99211 d. 90471, 90721

d. 90471, 90721

Patient was seen today for regular hemodialysis. no problems reported, tolerated procedure well. 90935- hemodialysis procedure with single physician evaluation 90937- hemodialysis procedure required repeated evalution with or without substantial revision of dialysis prescription 90945- dialysis procedure other than hemodialysis (e.g., peritoneal dialysis, hemofiltration, or other continuous renal replacemtn therapies), with single physician evaluation 99354 prolonged physican service in the office or other outpatient setting requiring direct (face-to-face) contact beyond the usual service; first hour (list separately in addition to code for office or other outpatient evaluation and management service) a. 90937 b. 99354 c. 90945 d. 90935

d. 90935

the census taken at midnight on August 1 showed 99 patients remaining in the hospital. on august 2, four patients were admitted, there was one fetal death, one DOA, and seven patients were discharged. one of these patients was admitted in the morning and remained only 8 hours. how many inpatient service days were rendered on August 2? a. 94 b. 95 c. 96 d. 97

d. 97

gail smith has presented to the ER in a coma with injuries sustained in a motor vehicle accident. according to her sister, gail has had a recent medical history taken at the public health department. the physician on call is grateful that she can access this patient information using the area's a. EDMS system b. CPOE c. expert system d. RHIO

d. RHIO

Fred is recovering nicely, so he asks Dr. Jones if he can go home for the weekend. Dr Jones approves a two-night leave of absence (LOA). Chances are Fred is a patient in a. an acute care facility; his LOA will decrease the months average daily inpatient census b. a long-term care facility, his LOA will increase the month's percentage of occupancy c. an acute care facility; his LOA will increase the month's total discharge days d. a long term care facility; his LOA will decrease the month's total inpatient service days.

d. a long term care facility; his LOA will decrease the month's total inpatient service days

Parker has type 1 diabetes with hypertension taht is currently controlled with medication. parker was admitted through the ED for an emergency appendectomy. following surgery, the patient developed an infection at the wound site that was treated with antibiotics. Parker's principal diagnosis is the a. complications of hypertension b. comorbidity of the wound infection c. comorbidity of type 1 diabets d. acute appendicitis

d. acute appendicitis

joanie howell presents to dr. franklin requesting rhinoplast. because howell is covered by medicare, dr. franklin must provide howell with a. a notice of exclusion, because rhinoplasty is not a medicare covered service b. an advance beneficiary notice, because rhinoplasty is not a medicare covered service c. a notice of exclusion, because howell's rhinoplasty may not be medically necessary. d. an advance beneficiary notice, because howell's rhinoplasty may not be medically necessary

d. an advance beneficiary notice because howell's rhinoplasty may not be medically necessary

a patient was treated for meningitis at age 3 (15 years ago). the patient is now 18. the patient's attorney is requesting information on the admission. you tell the clerk the information is a. no longer available because your facility retains information for 10 years after the last patient visit b. available, but the attorney will have to obtain a court order before you will release it. c. available, but the patient's parents will have to sign a consent for your to release it d. available, and the patient may sign consent to release the information in the record

d. available, and the patient may sign consent to release the information in the record

Four patients were discharged from crestview hospital yesterday. a final progress note is an appropriate discharge summary for a. howard, who died within 24 hours after his admission for a second heart attack in 2 weeks. b. jackson, who had no comorbidities or complications during this admission for replacement of a pacemaker battery c. fieldstone, who was admitted just 15 days following a heart attack for the acute onset of chest pain. d. babson, who delivered a healthy 8-pound boy without complications for either mother or child, and was discharged within 36 hours of admission.

d. babson, who delivered a healthy 8-pound boy without complications for either mother or child, and was discharged within 36 hours of admission.

an 11-year old female is brought to the emergency room with a compound, comminuted fracture of the right tibia and fibula. her mother was very seriously injured in the same accident and is unconscious. what should be done a. nothing, until consent can be obtained from the nearest relative b. the mother can be treated under implied consent but not the child c. the hospital should quickly seek a court-appointed guardian for the child. d. both patients can be treated under implied consent.

d. both patients can be treated under implied consent.

stan works in an acute care general hospital, fran works for a skilled nursing facility, ann is employed at an assisted living facility, and dan works for a home care provider. which people are employed in facilities that may seek Joint commission accreditation? a. only stan and fran b. only stan and dan c. only fran and dan d. dan, stan, and fran

d. dan, stan, and fran

The facility's policy for physician's verbal orders in accordance with state law and regulations needs updating. the first area of investigation is the qualifications of those individuals who have been authorized to record verbal orders. for this information you will consult the a. policy and procedure manual b. hospital's quality management plan. c. data dictionary d. hospital bylaws, rules, and regulations

d. hospital bylaws, rule, and regulations.

Dr. Reed tried to explain wound care to Mr. Baker prior to discharge, but Bake (who is 104 and moderately senile) just could not seem to understand or remember what the doctor said. Mr. Baker's daughter was with him, so Dr. Reed explained Mr. Baker's aftercare to his daughter. Dr. Reed should document discharge instructions a. in the discharge summary b. on a patient instructions form signed by Dr. Reed and Mr. Baker and filed in Mr. Baker's medical record. c. in the discharge summary and on a patient instructions form signed by Dr. Reed and Mr. Baker and filed in Baker's medical record. d. in the discharge summary and on a patient instructions form signed by Dr. Reed and Mr. Baker's daughter and filed in Mr. Baker's medical record.

d. in the discharge summary and on a patient instructions form signed by Dr. Reed and Mr. Baker's daughter and filed in Mr. Baker's medical record

which of the following diagnoses or procedures would prevent the normal delivery code, O80, from being assigned? a. occiput presentation b. single liveborn c. episiotomy d. low forceps

d. low forceps

Collins Family Hospital had a bed count of 150 for the first 6 months of the year. on june 1, it added 15 beds when it opened a new wing. if you are given the average length of stay for the year, can you calculate the annual bed turnover rate? how? a. yes, using the direct method b. yes, using the indirect method. c. yes, using the joint commision method. d. no, there is insufficient data to complete the calculation

d. no, there is insufficient data to complete the calculation

the formula used to calculate the percentage of ambulatory care visits made with same day appointments is a. number of patients seen with same day appointments for a period x 100 divided by the number of patients seen with addvance appointments for the same period b. number of patients seen with advance appointments for a period x 100 divided by the number of patients seen with the same day appointments for the same period c. number of patients seen with same day appointments for a period x 100 divided by the number of patients seen in the same period d. number of patients seen with advance apointments for a period x 100 divided by the number of patients seen in the same period

d. number of patients seen with same day appointments for a period x 100 divided by the number of patients seen in the same period

in preparation for an EHR, you are conducting a total facility inventory of all forms currently used. you must name each for for bar coding and indexing. the unnamed document in front of you includes a checklist for assessing an obstetric patient's lochia, fundus, and perineum. the document type you give to this form is. a. prenatal record b. labor record c. delivery room record d. postpartum record

d. postpartum record.

which of the following would be coded as a poisoning a. coumadin intoxication due to a cumulative effect b. idisoyncratic reaction to artane c. interaction between aldomet and a vasodilating agent d. reaction between coumadin and an over-the-counter medication

d. reaction between coumadin and an over-the-counter medication

as the coding supervisor, your job description includes working with agents who have been charged with detecting and correcting over payments made to your hospital in the Medicare Fee for Service program. you will need to develop a professional relationship with a. the OIG b. MEDPAR representatives c. QIO physicians d. recovery audit contractors

d. recovery audit contractors

employing the SOAP style of progress notes, choose the "assessment" statement from the following a. patient states low back pain with sciatica is as severe as it was on admission b. patient moving about very cautiously appears to be in pain c. adjust pain medication; begin physical therapy tomorrow. d. sciatica unimproved with hot pack therapy

d. sciatica unimproved with hot pack therapy

after your coders helped you rank the reasons for coding errors in the order of their importance, you then plotted the results on the chart above. the results of your work surprise you because a. you expected the coders to put more emphasis on time pressure b. you thought limited training was the primary reason for the errors c. the rankings show surprising disagreement on the issue d. the results appear to violate the Pareto Principle

d. the results appear to violate the Pareto principle

as the information security office at your facility, you have been asked to provide examples of the physical safeguards used to manage data security measures throughout the organization. which of the following would you provided. a. audit controls b. entity authentication c. chain-of-trust partner agreements d. workstation use and location

d. workstation use and location

a union campaign is being conducted at your facility. as a department manager, it is appropriate for you to tell employees that a. a strike is inevitable if the union wins b. wages will increase if the union is defeated c. you need the names of those involved in union activities d. you are opposed to the union

d. you are opposed to the union


Kaugnay na mga set ng pag-aaral

Management Chapter 3 Practice Questions

View Set

SEC and MSRB Rules Regarding Municipal Advisors

View Set

Standard Form, Vertex Form, and Intercept Form Quiz

View Set

Women's and Gender Studies Exam 2

View Set